Coherent solutions

This topic has expert replies
Legendary Member
Posts: 857
Joined: Wed Aug 25, 2010 1:36 am
Thanked: 56 times
Followed by:15 members

Coherent solutions

by AIM GMAT » Mon Apr 18, 2011 8:18 am
Coherent solutions for the problem of reducing health-care costs cannot be found within the current piecemeal system of paying these costs. The reason is that this system gives health-care providers and insurers every incentive to shift, wherever possible, the costs of treating illness onto each other or any other party, including the patient. That clearly is the lesson of the various reforms of the 1980s: push in on one part of this pliable spending balloon and an equally expensive bulge pops up elsewhere. For example, when the government health-care insurance program for the poor cut costs by disallowing payments for some visits to physicians, patients with advanced illness later presented themselves at hospital emergency rooms in increased numbers.

The argument proceeds by

(A) showing that shifting costs onto the patient contradicts the premise of health-care reimbursement
(B) attributing without justification fraudulent intent to people
(C) employing an analogy to characterize interrelationships
(D) denying the possibility of a solution by disparaging each possible alternative system
(E) demonstrating that cooperation is feasible by citing an instance
Thanks & Regards,
AIM GMAT

User avatar
Legendary Member
Posts: 582
Joined: Tue Mar 08, 2011 12:48 am
Thanked: 61 times
Followed by:6 members
GMAT Score:740

by force5 » Mon Apr 18, 2011 1:33 pm
This one was more dangerous than a 10 pound plutonium bomb.

stuck between C and E.

If i understand "argument proceeds by" means argument structure in terms of analogy.

Experts please advice if incorrect.

User avatar
Legendary Member
Posts: 1101
Joined: Fri Jan 28, 2011 7:26 am
Thanked: 47 times
Followed by:13 members
GMAT Score:640

by HSPA » Mon Apr 18, 2011 3:50 pm
2.42min...I went for B

My guess for the question "the argument proceeds by assuming what?"
First take: 640 (50M, 27V) - RC needs 300% improvement
Second take: coming soon..
Regards,
HSPA.

Master | Next Rank: 500 Posts
Posts: 184
Joined: Sat Apr 14, 2007 9:23 am
Location: Madison, WI
Thanked: 17 times

by ldoolitt » Mon Apr 18, 2011 6:11 pm
This is a method of reasoning question (the stem "the argument proceeds by" is always indicative of this) and I would almost bet my pants this is an LSAT question because of the complexity of the reasoning and the wording of the answer choices. I don't know that I've seen a method of reasoning this complex from a GMAT source, but then again I haven't done THAT many GMAT questions :). At any rate I'll give it a stab at the risk of being totally off base.

c: solution to health-care costs cannot be found within current system
p: because the current system gives p+i incentive to shift cost to others
p: a fall in one part will cause a rise in another
p: an example to show the up down relationship

(A) health care reimbursement is not mentioned in the problem (remember that "method of reasoning" falls under the prove family and thus, any information or event described in the answer that is not in the stimulus makes the answer incorrect)

(B) again this passage mentions nothing about fradulent intent. its not claiming that what the health care companies or patients are doing is fradulent.

(C) this is basically the last premise above so that would work

(D) the argument actually only mentions one "system" which is to say the current one. its not an ad nausem argument against each possible system that could exist

(E) the argument does cite an instance but that instance certainly doesn't demonstrate feasibility

User avatar
Master | Next Rank: 500 Posts
Posts: 234
Joined: Tue Feb 22, 2011 5:02 am
Thanked: 5 times
Followed by:3 members

by champmag » Mon Apr 18, 2011 10:29 pm
I go for C; However, we can use expert help in this question.

GMAT Instructor
Posts: 1302
Joined: Mon Oct 19, 2009 2:13 pm
Location: Toronto
Thanked: 539 times
Followed by:164 members
GMAT Score:800

by Testluv » Mon Apr 18, 2011 11:16 pm
As the above poster points out this is a method of argument question, which is fairly rare on the GMAT although you're more likely to get a question like than some crazy-heavy formal logic LSAT question.

The argument "proceeds by" employing an analogy--pressing on one part of a balloon causes another to bulge out--to characterize the interrelationships among the different parts of the healthcare system: if you cut costs in one area of healthcare, costs will increase in another. So, choice [spoiler](C)[/spoiler] is definitely correct.
Kaplan Teacher in Toronto

Legendary Member
Posts: 857
Joined: Wed Aug 25, 2010 1:36 am
Thanked: 56 times
Followed by:15 members

by AIM GMAT » Tue Apr 19, 2011 12:42 am
Thanks Testluv for the explanation , i tripped at this question .You are right , this is LSAT question which is quite unlikely to appear at GMAT . Trying to solve some heavy logic questions for following reasons :-

1] Working out the logic muscles .
2] Solving hard question to get an ease at GMAT CR and RC as well .
3] LSAT CRs are longer and more time consuming , hence that would help on GMAT front i.e to work on accuracy and timing .

Correct me if i am going in wrong direction .
Thanks & Regards,
AIM GMAT

GMAT Instructor
Posts: 1302
Joined: Mon Oct 19, 2009 2:13 pm
Location: Toronto
Thanked: 539 times
Followed by:164 members
GMAT Score:800

by Testluv » Tue Apr 19, 2011 7:40 am
Hi AimGmat,

actually what I was trying to say is this question is actually not as bad some of the others I see posted. You may actually get a question like this.

Regarding practicing on LSAT questions that are heavily grounded in formal logic: your reasons are good but you have to ask yourself whether your time would be better spent doing more GMATesque questions. And, LSAT questions that rely too heavily on formal logic really are outside the scope of the GMAT.

If you're going to practice with LSAT questions, practice on these types:

--strengthen/weaken (far less likely to contain formal logic and many of the patterns that appear in this question type overlap on the 2 tests)
--assumption
--role of statement (like boldface in GMAT)

But you should be careful with inference questions (LSAT likes to use a lot of formal logic in inference), and you shouldn't at all bother with all the LSAT-specific questions.

If you want to know more of what I think on the matter, just ask, and I'll try digging up some of my older more extensive posts on the topic.
Kaplan Teacher in Toronto